Difference between revisions of "2007 AIME II Problems/Problem 14"
(→Solution) |
|||
Line 14: | Line 14: | ||
Thus either <math>4x^6+4x^4+x^2+1=(4x^4+1)(x^2+1)</math> is 0 for any <math>x</math>, or <math>g(x)</math> satisfies the same constraints as <math>f(x)</math>. Continuing, by infinite descent, <math>f(x) = (x^2 + 1)^n</math> for some <math>n</math>. | Thus either <math>4x^6+4x^4+x^2+1=(4x^4+1)(x^2+1)</math> is 0 for any <math>x</math>, or <math>g(x)</math> satisfies the same constraints as <math>f(x)</math>. Continuing, by infinite descent, <math>f(x) = (x^2 + 1)^n</math> for some <math>n</math>. | ||
− | Since <math>f(2)+f(3)=125=5^n+10^n</math> for some <math>n</math>, we have <math>n=2</math>; so <math>f(5) = 676</math>. | + | Since <math>f(2)+f(3)=125=5^n+10^n</math> for some <math>n</math>, we have <math>n=2</math>; so <math>f(5) = \boxed{676}</math>. |
Revision as of 14:50, 25 June 2017
Problem
Let be a polynomial with real coefficients such that and for all , Find
Solution
Let be a root of . Then we have ; since is a root, we have ; therefore is also a root. Thus, if is real and non-zero, , so has infinitely many roots. Since is a polynomial (thus of finite degree) and is nonzero, has no real roots.
Note that is not constant. We then find two complex roots: . We find that , and that . This means that . Thus, are roots of the polynomial, and so will be a factor of the polynomial. (Note: This requires the assumption that . Clearly, , because that would imply the existence of a real root.)
The polynomial is thus in the form of . Substituting into the given expression, we have
Thus either is 0 for any , or satisfies the same constraints as . Continuing, by infinite descent, for some .
Since for some , we have ; so .
Comment:
The answer is clearly correct, but the proof has a gap, i.e. there is no reason that . Since has no real roots, the degree must be even. Consider . Then since is non-zero, . Now the function applied repeatedly from some real starting value of x becomes arbitrarily large, and the limit of as approaches infinity is 1, so =1 for all x, or . Then for some polynomial , and . Now suppose h has degree m. It is clearly monic. Assume that the next highest non-zero coefficient in h is k. Then, subtracting from both sides of the equation yields a polynomial equality with degree on the left and degree on the right, a contradiction. So , and .
See also
2007 AIME II (Problems • Answer Key • Resources) | ||
Preceded by Problem 13 |
Followed by Problem 15 | |
1 • 2 • 3 • 4 • 5 • 6 • 7 • 8 • 9 • 10 • 11 • 12 • 13 • 14 • 15 | ||
All AIME Problems and Solutions |
The problems on this page are copyrighted by the Mathematical Association of America's American Mathematics Competitions.